Autor Tema: Sistemas no inerciales rotantes.

0 Usuarios y 1 Visitante están viendo este tema.

16 Octubre, 2020, 02:13 am
Respuesta #20

jorge_nunez

  • $$\Large \color{#6a84c0}\pi$$
  • Mensajes: 43
  • País: ar
  • Karma: +0/-0

No es correcto.
En la primera no es cero la aceleración aunque no cambie el radio, pero si existe respecto del sistema rotatorio una aceleración centrípeta , ya que la masa gira circularmente respecto a este con velocidad angular \( \theta ' \) además te falta añadir la fuerza de Coriolis que es radial.

La segunda ecuación, dimensionalmente esta mal mezclas resta de ángulo con longitud del resorte y al segundo término de la igualdad le falta el radio, además le sobra el término de la fuerza de Coriolis que es radial.

Saludos.

Ya entendí. La segunda ecuación la voy a tener que pensar un poco más, porque el resorte se estira en una curva y en este momento estoy muy cansado como para pensar cómo hacerlo, pero ya entendí el error.

Gracias.

16 Octubre, 2020, 04:27 am
Respuesta #21

jorge_nunez

  • $$\Large \color{#6a84c0}\pi$$
  • Mensajes: 43
  • País: ar
  • Karma: +0/-0
Se me olvidaba:

Por la segunda ley de Newton:
\( \hat{r}: mΩ^2r-mgcos(θ)-N=0 \) (es igual a 0 porque el radio no cambia y \( r''=0 \))
\( \hat{θ}: mγr+k(l_o-θ)-2mΩrθ'=mθ'' \)
\( \hat{z}: 0 \) (no hay ninguna fuerza actuando en este eje)

Creo que esto es correcto.
No es correcto.
En la primera no es cero la aceleración aunque no cambie el radio, pero si existe respecto del sistema rotatorio una aceleración centrípeta , ya que la masa gira circularmente respecto a este con velocidad angular \( \theta ' \) además te falta añadir la fuerza de Coriolis que es radial.

La segunda ecuación, dimensionalmente esta mal mezclas resta de ángulo con longitud del resorte y al segundo término de la igualdad le falta el radio, además le sobra el término de la fuerza de Coriolis que es radial.

Saludos.

A ver entonces si entendí bien.

1. La primera ecuación tiene aceleración centrífuga para permitir el movimiento circular de la bolita. Tendría que igualarla a \( mR(θ')^2 \). Además, tengo que agregar la fuerza de Coriolis.
2. En la segunda tengo que reemplazar \( θ \) por \( θR \) en la parte del resorte y \( mθ'' \) por \( mRθ'' \), y sacar el termino de coriolis.

Una vez hechas esas correcciones, el problema debería salir.

16 Octubre, 2020, 07:07 pm
Respuesta #22

robinlambada

  • Moderador Global
  • Mensajes: 4,093
  • País: es
  • Karma: +0/-0
  • Sexo: Masculino
Se me olvidaba:

Por la segunda ley de Newton:
\( \hat{r}: mΩ^2r-mgcos(θ)-N=0 \) (es igual a 0 porque el radio no cambia y \( r''=0 \))
\( \hat{θ}: mγr+k(l_o-θ)-2mΩrθ'=mθ'' \)
\( \hat{z}: 0 \) (no hay ninguna fuerza actuando en este eje)

Creo que esto es correcto.
No es correcto.
En la primera no es cero la aceleración aunque no cambie el radio, pero si existe respecto del sistema rotatorio una aceleración centrípeta , ya que la masa gira circularmente respecto a este con velocidad angular \( \theta ' \) además te falta añadir la fuerza de Coriolis que es radial.

La segunda ecuación, dimensionalmente esta mal mezclas resta de ángulo con longitud del resorte y al segundo término de la igualdad le falta el radio, además le sobra el término de la fuerza de Coriolis que es radial.

Saludos.

A ver entonces si entendí bien.

1. La primera ecuación tiene aceleración centrífuga para permitir el movimiento circular de la bolita. Tendría que igualarla a \( mR(θ')^2 \). Además, tengo que agregar la fuerza de Coriolis.
2. En la segunda tengo que reemplazar \( θ \) por \( θR \) en la parte del resorte y \( mθ'' \) por \( mRθ'' \), y sacar el termino de coriolis.

Una vez hechas esas correcciones, el problema debería salir.
Correcto.

Saludos.
Envejecer es como escalar una gran montaña: mientras se sube las fuerzas disminuyen, pero la mirada es más libre, la vista más amplia y serena.

La verdadera juventud una vez alcanzada, nunca se pierde.

16 Octubre, 2020, 07:37 pm
Respuesta #23

jorge_nunez

  • $$\Large \color{#6a84c0}\pi$$
  • Mensajes: 43
  • País: ar
  • Karma: +0/-0
Muchísimas gracias a todos los que respondieron. Voy a intentar hacer el ejercicio hoy para asegurarme que no me quede ninguna duda.

Saludos.

16 Octubre, 2020, 07:44 pm
Respuesta #24

robinlambada

  • Moderador Global
  • Mensajes: 4,093
  • País: es
  • Karma: +0/-0
  • Sexo: Masculino
Pero tienes otro pequeño error que acabo de advertir en la ecuación de fuerzas tangenciales, ojo que la fuerza elástica se opone a la fuerza inercial del aro girando  \( mγr \), por tanto si el aro gira de forma horaria el muelle se encoje de su posición inicial debida a la fuerza de inercia  \( mγr \) en sentido antihorario y el muelle para restablecer el equilibrio de fuerzas se opone con una fuerza elástica en sentido horario, y al revés si el aro gira en sentido anti horario, en todo caso la fuerza  \( mγr \) y \( k(l_o-rθ) \) tienen sentidos opuestos y por tanto signos opuestos.

Si pones  \( k(l_o-rθ) \) das a entender que el muelle se encoje, por tanto asumes giro horario, en caso de anti horario sería  \( k(l_o+rθ) \)

Además siempre \( \vec{\Omega} \) y \( \vec{\theta} \), tienen sentido de giro contrarios.

Por ese pequeño detalle delmar cometió un desliz en:

\( \vec{r(t)}=rsen(\psi+\theta)\vec{i}+rcos(\psi+\theta)\vec{j} \)

La velocidad, considerando \( \psi'=\Omega \)

\( \vec{r'(t)}=rcos(\psi+\theta)(\Omega+\theta')\vec{i}-rsen(\psi-\theta)(\Omega-\theta')\vec{j} \)


\( \vec{r(t)}=rsen(\psi-\theta)\vec{i}+rcos(\psi-\theta)\vec{j} \)

Esto sería así si \( \psi \) mide el ángulo de giro del aro desde posición la de equilibrio del resorte en el instante inicial hasta la posición de equilibrio del resorte en un instante t.

\( \vec{r'(t)}=rcos(\psi-\theta)(\Omega-\theta')\vec{i}-rsen(\psi-\theta)(\Omega-\theta')\vec{j} \)

En todo caso siempre las velocidades angulares se oponen.

Envejecer es como escalar una gran montaña: mientras se sube las fuerzas disminuyen, pero la mirada es más libre, la vista más amplia y serena.

La verdadera juventud una vez alcanzada, nunca se pierde.

16 Octubre, 2020, 08:10 pm
Respuesta #25

robinlambada

  • Moderador Global
  • Mensajes: 4,093
  • País: es
  • Karma: +0/-0
  • Sexo: Masculino
Una última cosa, no quiero ser pesado, pero el enunciado no es nada claro, no te dice nada del plano de giro del aro.

¿Estas seguro que la gravedad influye? es decir, ¿estas seguro que gira verticalmente ? o ¿puede girar el aro en un plano horizontal? si es esto último se simplifica los puntos de equilibrio, ya que la gravedad se anula con la componente normal y no hay movimiento verticalmente.

Para los puntos de equilibrio en este caso, tendrías que ver que en dirección \( \hat{\theta} \) si no hay aceleración en el sistema no inercial, la fuerza elástica se iguala a la de inercia y la velocidad también es cero ya que la fuerza de inercia es constante y al serlo también la elástica, quiere decir que el resorte no se estira ni se encoge , por ello la masa se halla en reposo respecto al S.R. rotatorio y desaparece la aceleración de Coriolis.


En otro caso es bastante más complejo ya que el resorte se estiraría y encogería en función de si la masa sube o baja ( suponemos el aro en un plano vertical ) lo que si ocurriría a largo plazo , es que la fuerza de inercia de la aceleración del aro se compensa con la fuerza del resorte que se opone, se podría considerar esta nueva posición del resorte como un nuevo punto de "equilibrio" del resorte ( con una nueva \(  l_o \)) y a partir de entonces, solo afectarían la fuerza elástica y la componente tangencial de la gravitatoria que se oponen , produciendo un movimiento oscilante circular respecto al sistema no inercial. Pero ya digo que este caso es más complejo.
Envejecer es como escalar una gran montaña: mientras se sube las fuerzas disminuyen, pero la mirada es más libre, la vista más amplia y serena.

La verdadera juventud una vez alcanzada, nunca se pierde.

16 Octubre, 2020, 10:02 pm
Respuesta #26

jorge_nunez

  • $$\Large \color{#6a84c0}\pi$$
  • Mensajes: 43
  • País: ar
  • Karma: +0/-0
Una última cosa, no quiero ser pesado, pero el enunciado no es nada claro, no te dice nada del plano de giro del aro.

¿Estas seguro que la gravedad influye? es decir, ¿estas seguro que gira verticalmente ? o ¿puede girar el aro en un plano horizontal? si es esto último se simplifica los puntos de equilibrio, ya que la gravedad se anula con la componente normal y no hay movimiento verticalmente.

Para los puntos de equilibrio en este caso, tendrías que ver que en dirección \( \hat{\theta} \) si no hay aceleración en el sistema no inercial, la fuerza elástica se iguala a la de inercia y la velocidad también es cero ya que la fuerza de inercia es constante y al serlo también la elástica quiere decir que el resorte no se estira ni se encoge , por ello la masa se halla en reposo respecto al S.R. rotatorio y desaparece la aceleración de Coriolis.


En otro caso es bastante más complejo ya que el resorte se estiraría y encogería en función de si la masa sube o baja ( suponemos el aro en un plano vertical ) lo que si ocurriría a largo plazo , es que la fuerza de inercia de la aceleración del aro se compensa con la fuerza del resorte que se opone, se podría considerar esta nueva posición del resorte como un nuevo punto de "equilibrio" del resorte ( con una nueva\(  l_o \)) y a partir de entonces, solo afectarían la fuerza elástica y la componente tangencial de la gravitatoria que se oponen , produciendo un movimiento oscilante circular respecto al sistema no inercial. Pero ya digo que este caso es más complejo.

No sos pesado. Al contrario, mientras más claro me quede el enunciado mejor.

Coincido en que el enunciado no es claro. En algunos ejercicios, anotan para dónde apunta la gravedad y no hay ambigüedad, pero en otros no. La verdad, asumí que era vertical sin pensarlo. Podría hacer el ejercicio como si fuera un plano horizontal y más adelante preguntale a los profesores de qué manera gira (no me acuerdo si lo dije, pero este cuatrimestre no están viendo sistemas rotantes con fuerza de Euler, asique tendría que consulatrlo por separado).

Muchas gracias

17 Octubre, 2020, 02:19 am
Respuesta #27

jorge_nunez

  • $$\Large \color{#6a84c0}\pi$$
  • Mensajes: 43
  • País: ar
  • Karma: +0/-0
Bueno a ver. Lo volví a hacer. Esta vez en un plano horizontal para simplificar las cosas. Si esta bien, lo puedo llegar a intentar con gravedad.

Primero, me di cuenta que la velocidad angular, en el sistema de referencia que elegí, es \( \bar{Ω}=-Ω\hat{z} \) y no positiva como había puesto antes.

Por otra parte, una duda con respecto a la parte del resorte y lo que anotaste sobre el signo. En sí, no tuve en cuenta la dirección de \( F_γ \), si no solamente el resorte. Lo que hago es \( \hat{r}: mγR+k(l_o-Rθ) \). Si el resorte se comprime, entonces \( (l_o-Rθ)>0 \) y la fuerza será positiva, lo que concuerda con el eje. Si \( (l_o-Rθ)<0 \), la fuerza elástica es negativa y también concuerda con el sistema de referencia.

En fin, ahora a lo que hice:

Ajunté una imagen con el nuevo DCL.

Inciso a): Lo que cambia acá es: \( \bar{Ω}=-Ω\hat{z} \), \( \bar{V}_{rot}=-Rθ'\hat{θ} \) y \( \bar{γ}=-γ\hat{z} \) (a lo que agrego que \( \bar{r}=R\hat{r} \)). Las fuerzas inerciales entonces son:

· \( F_{cent}=-m\bar{Ω}X(\bar{Ω}X\bar{r})=mΩ\hat{z}X(-Ω\hat{z}XR\hat{r})= \)\( mΩ^2R\hat{r} \)
· \( F_{cor}=-2m\bar{Ω}xV_{rot}=2mΩ\hat{z}X(-Rθ')\hat{θ}= \)\( 2mΩRθ'\hat{r} \)
· \( F_γ=-m\hat{γ}X\hat{r}=-m(-γ\hat{z)}XR\hat{r}= \)\( mγR\hat{θ} \)

Entonces, las ecuaciones de Newton quedan:

(1) \( \hat{r}: -mRθ'^2=mΩ^2R+2mΩRθ'-F_V \)
(2) \( \hat{θ}: mRθ''=mγR+kR(\frac{π}{2}-θ) \)
(3) \( \hat{z}: 0=N-mg \)

Inciso b): En los puntos de equilibrio (los llamo \( θ_{eq} \)), la aceleración se anula. Según la ecuación (2):

\( 0=mγR+kR(\frac{π}{2}-θ_{eq}) \)
\( 0=mγ+k\frac{π}{2}-kθ_{eq} \)
\( kθ_{eq}=mγ+k\frac{π}{2} \)

\( θ_{eq}=\frac{m}{k}γ+\frac{π}{2} \)

Para encontrar la ecuación de movimiento, uso la misma ecuación (2):

\( mθ''=mγ+k(\frac{π}{2}-θ) \)
\( mθ''=mγ+k\frac{π}{2}-kθ \)
\( mθ''+kθ=mγ+k\frac{π}{2} \)
\( θ''+\frac{k}{m}θ=γ+\frac{πk}{2m} \)

Llamo \( ω^2=\frac{k}{m} \). Una posible solución para esta ecuación diferencial es:

\( θ(t)=Acos(ωt+α_o)+\frac{m}{k}γ+\frac{π}{2} \) (me salteé la parte en la que busco la solución particular porque creo que es sencilla).

Y, por las condiciones iniciales (de nuevo, me salteo las cuentas, pero \( θ'(0)=0 \) y \( θ(0)=ϕ_o \), y sale).

\( θ(t)=(ϕ_o-\frac{m}{k}γ-\frac{π}{2})cos(ωt)+(\frac{m}{k}γ+\frac{π}{2}) \)

18 Octubre, 2020, 01:22 am
Respuesta #28

delmar

  • Moderador Global
  • Mensajes: 3,543
  • País: pe
  • Karma: +0/-0
  • Sexo: Masculino
Continuando con el análisis del problema, utilizando una referencia inercial XY, y  considerando que el aro  rota en un plano horizontal, paralelo a tierra. El esquema es el mismo, ahí se indica el sentido de rotación del aro, considerándolo positivo ; se esta suponiendo que \( \psi \) es el ángulo determinado por el extremo del resorte fijo al aro y \( \theta \) el ángulo determinado por el extremo móvil del resorte y el extremo fijo del resorte (arco que abraza el resorte), es obvio que el extremo móvil coincide con la bolita, los sentidos positivos de ambos ángulos estan mostrados en el esquema; en esas condiciones \( \psi \) es un ángulo respecto a la referencia inercial; mientras que \( \theta \) es un ángulo respecto a la referencia solidaria al aro; por ello la obtención de \( \theta(t) \) es la solución que se pide, aún cuando este dentro de  un análisis con referencia inercial, en esas circunstancias es válido, no entiendo bien la observación de robinlambada, en otras palabras \( \vec{r(t)} \) y \( \psi' \) son lo que he puesto  :

Hola

Las leyes de Newton, son válidas en las referencias inerciales, entiendo que bajo ciertas circunstancias se utilizan las referencias móviles. Un análisis para referencias inerciales lo muestro, adjunto un esquema :



La referencia XY es inercial, solidaria a la tierra.Hay un extremo del resorte solidario al aro y forma un ángulo \( \psi \) con la parte positiva del eje Y, la velocidad angular de su radio vector es igual a la velocidad angular del aro \( \psi'=\Omega \)
; mientras el resorte abraza un arco \( \theta \), en consecuencia la posición de la bolita queda determinada por el ángulo \( \psi+\theta \), su vector posición para un t genérico será :

\( \vec{r(t)}=rsen(\psi+\theta)\vec{i}+rcos(\psi+\theta)\vec{j} \)

La velocidad, considerando \( \psi'=\Omega \)

\( \vec{r'(t)}=rcos(\psi+\theta)(\Omega+\theta')\vec{i}-rsen(\psi+\theta)(\Omega+\theta')\vec{j} \)

Su aceleración

\( \vec{r''(t)}=r(-sen(\psi+\theta)(\Omega+\theta')^2+cos(\psi+\theta)(\gamma+\theta''))\vec{i}-r(cos(\psi+\theta)(\Omega+\theta')^2+sen(\psi+\theta)(\gamma+\theta''))\vec{j} \)

Por otro lado la suma de fuerzas (elástica y normal) será :

\( \vec{F}=N(sen(\psi+\theta)\vec{i}+cos(\psi+\theta)\vec{j})+k(l_0-r \theta)(cos(\psi+\theta)\vec{i}-sen(\psi+\theta)\vec{j}) \)

Aplicando la 2da Ley de Newton

\( \vec{F}=m\vec{r''(t)} \)


Continuando :

Dirección x

\( Nsen(\psi+\theta)+k(l_0-r \theta)cos(\psi+\theta)=mr \ (-sen(\psi+\theta)(\Omega+\theta')^2+cos(\psi+\theta)(\gamma+\theta'')) \) Ec. 1

Dirección y

\( Ncos(\psi+\theta)-k(l_0-r \theta)sen(\psi+\theta)=-mr \ (cos(\psi+\theta)(\Omega+\theta')^2+sen(\psi+\theta)(\gamma+\theta'')) \) Ec. 2

Multiplicando la Ec. 1 por \( cos(\psi+\theta) \) y la Ec. 2 por \( -sen(\psi+\theta) \) y sumando ambas ecuaciones se tiene :

\( k(l_0-r \theta)=mr(\gamma+\theta'') \)

Poniendo en la forma adecuada :

\( \theta''+(\frac{k}{m}) \ \theta=(\frac{\pi k}{2m}-\gamma) \)

Esto se puede resolver es la suma de la solución homogénea y una solución particular, considerando las condiciones iniciales :

\( \theta(0)=\theta_0=\frac{\pi}{2} \)

\( \theta'(0)=\Omega_0 \) tiene la misma velocidad angular, que el aro.

Terminaré de desarrollar el problema el lunes si Dios quiere.

Saludos

Nota 1: El sentido de rotación del aro, se ha considerado positivo, esto no es lo que usualmente se considera (positivo sentido antihorario) pero teniendo esta consideración los resultados son objetivos,


18 Octubre, 2020, 05:15 pm
Respuesta #29

Richard R Richard

  • Ingeniero Industrial
  • $$\Large \color{#5b61b3}\pi\,\pi\,\pi\,\pi\,\pi$$
  • Mensajes: 2,318
  • País: ar
  • Karma: +1/-0
  • Sexo: Masculino
  • Dentro de la ciencia todo,fuera de la ciencia nada
En un Principio plantee el sistema de referencia solidario a la base del resorte, pero veo que , lo han resuelto suponiendo el origen de coordenadas en el centro de la circunferencia, para lo cual el sistema es un SRnI acelerado angular mente con modulo iguala a la aceleración tangencial sobre el radio constante.
Desde ese sistema de referencia, lo que hay que explicar es el estiramiento y acortamiento del resorte, la falta de aceleración radial y en dirección al eje de giro es compensada por el vinculo del anillo sobre la circunferencia.


En resumidas cuentas la diferencia entre resolver con fuerzas ficticias  en un SRnI  y sin ellas en un SRI es un simple pasaje de términos, y en interpretarlos correctamente a cada uno.
Saludos  \(\mathbb {R}^3\)